LSAT and Law School Admissions Forum

Get expert LSAT preparation and law school admissions advice from PowerScore Test Preparation.

 Administrator
PowerScore Staff
  • PowerScore Staff
  • Posts: 8917
  • Joined: Feb 02, 2011
|
#32541
Complete Question Explanation

Strengthen—CE. The correct answer choice is (E)

In this stimulus, the author describes a hospital study in which magnets were used to treat people with severe joint pain. In the study, “76 percent of those who were treated with magnets reported reduced pain after just 3 weeks.” The implication of the study is a causal relationship in which the treatment with magnets can reduce arthritic pain.

Without providing much detail, the stimulus tells us that dogs and humans have similar physiologies. Based on this similarity, the author shifts to the idea that magnets likely can be used to relieve arthritic pain in older dogs. More specifically, the author concludes that a device called the Magno-Blanket, which brings magnets into the same proximity to the dog’s joints as they were to the patients’ joints in the hospital study, “is probably able to relieve arthritic pain in older dogs.”

This argument raises several questions that go unanswered. For example, what is the nature and extent of the similarity in the physiologies of dogs and humans? Were the humans in the hospital study treated using a device similar to the Magno-Blanket? If not, what are the differences between the magnets used in the study and the Magno-Blanket, and how might those differences affect the results?

These issues can clutter our prephrase and take our focus off of our standard approach to causal reasoning on the LSAT. The most important piece of the argument is the existence of a causal relationship between treatment using magnets and reduction in severe joint pain. Without that causal relationship, there is no reason to think the Magno-Blanket can do anything to help relieve arthritic pain in dogs. Focus your prephrase on this causal relationship, while at the same time being aware of the other logical gaps, any of which the correct answer choice might address.

Answer choice (A): Implying there is a physiological similarity between dogs and other pets, as suggested by this answer choice, does nothing to support the connection between dogs and humans.

Answer choice (B): The stimulus does not tell us about the transmission of messages from nerve cells to the brain, so it is unclear how this information is relevant to the treatment of joint pain in humans, let alone the suggestion that the Magno-Blanket would relieve arthritic pain in dogs.

Answer choice (C): This answer choice indicates there is a need for a new device that could relieve arthritic pain in dogs, but does not provide support for the conclusion that the Magno-Blanket would be effective.

Answer choice (D): This answer choice highlights some limitations on the effectiveness of treating humans with magnets, and does nothing to support the idea that the Magno-Blanket would be effective in treating dogs.

Answer choice (E): This is the correct answer choice, because it removes a potential alternate explanation for the results of the hospital study. The stimulus did not provide, as a fact, that treatment with magnets reduces joint pain. Rather, the author implicitly concluded that treatment with magnets relieved joint pain in humans. An alternate explanation for that result could have been that there was a placebo, or psychosomatic effect, at play. In other words, perhaps the idea of receiving the magnet therapy caused the test subjects to experience less pain, rather than the magnets themselves causing the result. By removing an alternate cause, the answer choice strengthens the author’s causal conclusion, which depends on the existence of a causal relationship between treatment with magnets and decreased joint pain.
 15veries
  • Posts: 113
  • Joined: Sep 25, 2016
|
#29492
Hi,

Isn't E a very weak strengthener?
It does not talk about the relationship discussed directly in any way...
 Adam Tyson
PowerScore Staff
  • PowerScore Staff
  • Posts: 5153
  • Joined: Apr 14, 2011
|
#29499
Thanks for the question, 15!

Two things to deal with here: First, a "weak" strengthen is still a strengthen, and if that's the best answer then that's enough. You don't have to help very much, just help! In this case, the other four answers are no help at all, so E must be the best answer no matter how poor a job you think it does. Get used to weak answers, and even bad answers, on this test - the instructions are clear in asking you to pick the BEST answer (of the five presented), rather than the RIGHT answer or a GOOD answer. "Best" is relative!

Second, what's happening in this answer is a classic Causal Reasoning - Strengthen answer. This answer tells us that when the cause (magnets) is absent, the effect (pain relief) is also absent. While that certainly doesn't prove a causal claim, it's one of the 5 standard ways to strengthen one, and should be a great match for your prephrase once you recognize the causality.

If you are having any troubles with causal reasoning, take another look at Lessons 3 and 4 in particular to see how to handle them.

Keep at it!
 15veries
  • Posts: 113
  • Joined: Sep 25, 2016
|
#29523
Hi, thanks for your reply.
I see, so always the best answer instead of good one...good lesson :)
 ssnasir
  • Posts: 22
  • Joined: Feb 22, 2020
|
#74444
Hi there,

E is the best answer because it addresses the causal relationship, but I was tempted by (D). Answer choice D was addressing the people who did not report reduced pain because they didn’t suffer from severe joint pain. So, it made sense that since they didn’t have severe pain the magnet would not have helped them suggesting that those with severe pain reported reduced pain because of the magnet. So, wouldn't this help the argument as well?
 Jeremy Press
PowerScore Staff
  • PowerScore Staff
  • Posts: 1000
  • Joined: Jun 12, 2017
|
#74463
Hi ssnasir,

I think the best reading of answer choice D is that it is a weaken answer. To get where you're trying to go (to see answer choice D as a weak strengthener), you'd need the answer to say that "[t]he patients in the hospital study suffering from severe joint pain who, after being treated with magnets, reported a smaller degree of reduced pain tended not to be those suffering from the most severe pain." If that were the case, we might think the magnets affect everyone, and have a sliding scale effect depending upon how much pain a patient has to start. In that case, it does at least weakly increase my faith in the ability of magnets to reduce pain.

But, as worded, answer choice D states that some of the patients with severe pain are not reporting reduced pain at all. That makes me doubt the effectiveness of magnets on at least a certain subset of patients, and introduces some doubt about the (rather generally worded) conclusion that the blanket is "probably able to relieve arthritic pain in older dogs [generally speaking]."

I hope this helps!

Jeremy
 ssnasir
  • Posts: 22
  • Joined: Feb 22, 2020
|
#74536
Thank you!

Get the most out of your LSAT Prep Plus subscription.

Analyze and track your performance with our Testing and Analytics Package.